Violympic toán 9

Phan Tiến Nhật

Cho a, b, c là các số thực dương tùy ý. Chứng minh rằng:

\(\left(a^3+b^3+c^3\right)\left(\frac{1}{a^3}+\frac{1}{b^3}+\frac{1}{c^3}\right)\ge\frac{3}{2}\left(\frac{a+b}{c}+\frac{b+c}{a}+\frac{c+a}{b}\right)\)


Các câu hỏi tương tự
fghj
Xem chi tiết
bt ko
Xem chi tiết
Nguyễn Đức Anh
Xem chi tiết
Phác Chí Mẫn
Xem chi tiết
Đại Ngọc
Xem chi tiết
Lê Đình Quân
Xem chi tiết
Nguyễn Thanh Hiền
Xem chi tiết
Hoàng Quốc Tuấn
Xem chi tiết
Hoàng Quốc Tuấn
Xem chi tiết